If a sequence $s_n$ of real numbers converges to $s>a$ then $s_n>a$Prove that the convergence of the sequence ($s_n$) implies the convergence of ($s_n^3$)If a sequence $s_n$ converges, does $f(s_n)$ converge?Simple sequence convergence question…Conclude that if all but finitely many $s_n$ belong to $[a, b]$, lim $s_n$ belongs to $[a,b]$Simple proof that this sequence converges [verification]Show that if $(sum x_n)$ converges absolutely and $(y_n)$ is bounded then $(sum x_n y_n)$ convergesShow that if all convergent sub-sequences of a sequence $s_n$ converge to 0 and $s_n$ is bounded, then $s_n$ converges to $0$Sequence of divisions such that its upper and lower function limits are equalIf sequence $s_n$ converges, then sequence $$ converges as well(check the logic)Show directly that if $s_n$ is a Cauchy sequence then so is $$. Conclude that $$ converges whenever $s_n$ converges.

The (Easy) Road to Code

Can I negotiate a patent idea for a raise, under French law?

Do Cubics always have one real root?

Short scifi story where reproductive organs are converted to produce "materials", pregnant protagonist is "found fit" to be a mother

What can I do if someone tampers with my SSH public key?

Is "cogitate" used appropriately in "I cogitate that success relies on hard work"?

What does the Digital Threat scope actually do?

Which country has more?

What does *dead* mean in *What do you mean, dead?*?

How can a demon take control of a human body during REM sleep?

School performs periodic password audits. Is my password compromised?

How can I portion out frozen cookie dough?

Help! My Character is too much for her story!

Are E natural minor and B harmonic minor related?

How to educate team mate to take screenshots for bugs with out unwanted stuff

Idiom for feeling after taking risk and someone else being rewarded

Will expression retain the same definition if particle is changed?

Why do we say 'Pairwise Disjoint', rather than 'Disjoint'?

Professor forcing me to attend a conference, I can't afford even with 50% funding

Did Amazon pay $0 in taxes last year?

Writing text next to a table

Why restrict private health insurance?

Strange opamp's output impedance in spice

Locked Away- What am I?



If a sequence $s_n$ of real numbers converges to $s>a$ then $s_n>a$


Prove that the convergence of the sequence ($s_n$) implies the convergence of ($s_n^3$)If a sequence $s_n$ converges, does $f(s_n)$ converge?Simple sequence convergence question…Conclude that if all but finitely many $s_n$ belong to $[a, b]$, lim $s_n$ belongs to $[a,b]$Simple proof that this sequence converges [verification]Show that if $(sum x_n)$ converges absolutely and $(y_n)$ is bounded then $(sum x_n y_n)$ convergesShow that if all convergent sub-sequences of a sequence $s_n$ converge to 0 and $s_n$ is bounded, then $s_n$ converges to $0$Sequence of divisions such that its upper and lower function limits are equalIf sequence $s_n$ converges, then sequence $$ converges as well(check the logic)Show directly that if $s_n$ is a Cauchy sequence then so is $$. Conclude that $$ converges whenever $s_n$ converges.













1












$begingroup$


Let $s_n$ be a sequence of real numbers that converges to $s>a$. Then there exists an $N>0$ such that $n>Nimplies s_n>a$.



PROOF:
By definition of convergence in sequences we have that for all $varepsilon>0$, there exists $N>0$ such that $n>N$ implies $|s_n-s|<varepsilon$.



Choose $varepsilon=s-a$. Then $|s_n-s|<s-a$. By properties of absolute values and inequalities we have $-s+a<s_n-s<s-a$ which implies $s_n>a$.



Is my proof rigorous, correct? I'd appreciate any comments for improvement.










share|cite|improve this question











$endgroup$











  • $begingroup$
    Looks okay to me, do you know where you used the condition $s>a$ or did you even use it?
    $endgroup$
    – kingW3
    yesterday











  • $begingroup$
    @kingW3 I think implicitly I assumed that $s-a>0$ since $varepsilon>0$. Is that okay to do?
    $endgroup$
    – johnny09
    yesterday






  • 1




    $begingroup$
    It is, you're able to use $epsilon = s-a$ because $s-a>0iff s>a$. You could mention it explicitly like "Choose $epsilon=s-a>0$" but you don't really need to. I just wanted to check whether you understood that.
    $endgroup$
    – kingW3
    yesterday















1












$begingroup$


Let $s_n$ be a sequence of real numbers that converges to $s>a$. Then there exists an $N>0$ such that $n>Nimplies s_n>a$.



PROOF:
By definition of convergence in sequences we have that for all $varepsilon>0$, there exists $N>0$ such that $n>N$ implies $|s_n-s|<varepsilon$.



Choose $varepsilon=s-a$. Then $|s_n-s|<s-a$. By properties of absolute values and inequalities we have $-s+a<s_n-s<s-a$ which implies $s_n>a$.



Is my proof rigorous, correct? I'd appreciate any comments for improvement.










share|cite|improve this question











$endgroup$











  • $begingroup$
    Looks okay to me, do you know where you used the condition $s>a$ or did you even use it?
    $endgroup$
    – kingW3
    yesterday











  • $begingroup$
    @kingW3 I think implicitly I assumed that $s-a>0$ since $varepsilon>0$. Is that okay to do?
    $endgroup$
    – johnny09
    yesterday






  • 1




    $begingroup$
    It is, you're able to use $epsilon = s-a$ because $s-a>0iff s>a$. You could mention it explicitly like "Choose $epsilon=s-a>0$" but you don't really need to. I just wanted to check whether you understood that.
    $endgroup$
    – kingW3
    yesterday













1












1








1





$begingroup$


Let $s_n$ be a sequence of real numbers that converges to $s>a$. Then there exists an $N>0$ such that $n>Nimplies s_n>a$.



PROOF:
By definition of convergence in sequences we have that for all $varepsilon>0$, there exists $N>0$ such that $n>N$ implies $|s_n-s|<varepsilon$.



Choose $varepsilon=s-a$. Then $|s_n-s|<s-a$. By properties of absolute values and inequalities we have $-s+a<s_n-s<s-a$ which implies $s_n>a$.



Is my proof rigorous, correct? I'd appreciate any comments for improvement.










share|cite|improve this question











$endgroup$




Let $s_n$ be a sequence of real numbers that converges to $s>a$. Then there exists an $N>0$ such that $n>Nimplies s_n>a$.



PROOF:
By definition of convergence in sequences we have that for all $varepsilon>0$, there exists $N>0$ such that $n>N$ implies $|s_n-s|<varepsilon$.



Choose $varepsilon=s-a$. Then $|s_n-s|<s-a$. By properties of absolute values and inequalities we have $-s+a<s_n-s<s-a$ which implies $s_n>a$.



Is my proof rigorous, correct? I'd appreciate any comments for improvement.







real-analysis proof-verification convergence






share|cite|improve this question















share|cite|improve this question













share|cite|improve this question




share|cite|improve this question








edited yesterday









Shaun

9,334113684




9,334113684










asked yesterday









johnny09johnny09

673624




673624











  • $begingroup$
    Looks okay to me, do you know where you used the condition $s>a$ or did you even use it?
    $endgroup$
    – kingW3
    yesterday











  • $begingroup$
    @kingW3 I think implicitly I assumed that $s-a>0$ since $varepsilon>0$. Is that okay to do?
    $endgroup$
    – johnny09
    yesterday






  • 1




    $begingroup$
    It is, you're able to use $epsilon = s-a$ because $s-a>0iff s>a$. You could mention it explicitly like "Choose $epsilon=s-a>0$" but you don't really need to. I just wanted to check whether you understood that.
    $endgroup$
    – kingW3
    yesterday
















  • $begingroup$
    Looks okay to me, do you know where you used the condition $s>a$ or did you even use it?
    $endgroup$
    – kingW3
    yesterday











  • $begingroup$
    @kingW3 I think implicitly I assumed that $s-a>0$ since $varepsilon>0$. Is that okay to do?
    $endgroup$
    – johnny09
    yesterday






  • 1




    $begingroup$
    It is, you're able to use $epsilon = s-a$ because $s-a>0iff s>a$. You could mention it explicitly like "Choose $epsilon=s-a>0$" but you don't really need to. I just wanted to check whether you understood that.
    $endgroup$
    – kingW3
    yesterday















$begingroup$
Looks okay to me, do you know where you used the condition $s>a$ or did you even use it?
$endgroup$
– kingW3
yesterday





$begingroup$
Looks okay to me, do you know where you used the condition $s>a$ or did you even use it?
$endgroup$
– kingW3
yesterday













$begingroup$
@kingW3 I think implicitly I assumed that $s-a>0$ since $varepsilon>0$. Is that okay to do?
$endgroup$
– johnny09
yesterday




$begingroup$
@kingW3 I think implicitly I assumed that $s-a>0$ since $varepsilon>0$. Is that okay to do?
$endgroup$
– johnny09
yesterday




1




1




$begingroup$
It is, you're able to use $epsilon = s-a$ because $s-a>0iff s>a$. You could mention it explicitly like "Choose $epsilon=s-a>0$" but you don't really need to. I just wanted to check whether you understood that.
$endgroup$
– kingW3
yesterday




$begingroup$
It is, you're able to use $epsilon = s-a$ because $s-a>0iff s>a$. You could mention it explicitly like "Choose $epsilon=s-a>0$" but you don't really need to. I just wanted to check whether you understood that.
$endgroup$
– kingW3
yesterday










1 Answer
1






active

oldest

votes


















2












$begingroup$

I would just add, after “Choose $varepsilon=s-a$”, that there is a $Ninmathbb N$ such that $n>Nimplieslvert s_n-srvert<varepsilon$.






share|cite|improve this answer









$endgroup$












  • $begingroup$
    Something like: "Let $varepsilon=s-a>0$. Then, by definition of limit of sequence, there exists $N in mathbbN$ such that $|s_n-s|<varepsilon=s-a$ whenever $n>N$. By properties of absolute values and inequalities we have $-s+a<s_n-s$ which implies $s_n>a$ for $n>N$."
    $endgroup$
    – md2perpe
    yesterday











  • $begingroup$
    That would be just fine!
    $endgroup$
    – José Carlos Santos
    yesterday










Your Answer





StackExchange.ifUsing("editor", function ()
return StackExchange.using("mathjaxEditing", function ()
StackExchange.MarkdownEditor.creationCallbacks.add(function (editor, postfix)
StackExchange.mathjaxEditing.prepareWmdForMathJax(editor, postfix, [["$", "$"], ["\\(","\\)"]]);
);
);
, "mathjax-editing");

StackExchange.ready(function()
var channelOptions =
tags: "".split(" "),
id: "69"
;
initTagRenderer("".split(" "), "".split(" "), channelOptions);

StackExchange.using("externalEditor", function()
// Have to fire editor after snippets, if snippets enabled
if (StackExchange.settings.snippets.snippetsEnabled)
StackExchange.using("snippets", function()
createEditor();
);

else
createEditor();

);

function createEditor()
StackExchange.prepareEditor(
heartbeatType: 'answer',
autoActivateHeartbeat: false,
convertImagesToLinks: true,
noModals: true,
showLowRepImageUploadWarning: true,
reputationToPostImages: 10,
bindNavPrevention: true,
postfix: "",
imageUploader:
brandingHtml: "Powered by u003ca class="icon-imgur-white" href="https://imgur.com/"u003eu003c/au003e",
contentPolicyHtml: "User contributions licensed under u003ca href="https://creativecommons.org/licenses/by-sa/3.0/"u003ecc by-sa 3.0 with attribution requiredu003c/au003e u003ca href="https://stackoverflow.com/legal/content-policy"u003e(content policy)u003c/au003e",
allowUrls: true
,
noCode: true, onDemand: true,
discardSelector: ".discard-answer"
,immediatelyShowMarkdownHelp:true
);



);













draft saved

draft discarded


















StackExchange.ready(
function ()
StackExchange.openid.initPostLogin('.new-post-login', 'https%3a%2f%2fmath.stackexchange.com%2fquestions%2f3140507%2fif-a-sequence-s-n-of-real-numbers-converges-to-sa-then-s-na%23new-answer', 'question_page');

);

Post as a guest















Required, but never shown

























1 Answer
1






active

oldest

votes








1 Answer
1






active

oldest

votes









active

oldest

votes






active

oldest

votes









2












$begingroup$

I would just add, after “Choose $varepsilon=s-a$”, that there is a $Ninmathbb N$ such that $n>Nimplieslvert s_n-srvert<varepsilon$.






share|cite|improve this answer









$endgroup$












  • $begingroup$
    Something like: "Let $varepsilon=s-a>0$. Then, by definition of limit of sequence, there exists $N in mathbbN$ such that $|s_n-s|<varepsilon=s-a$ whenever $n>N$. By properties of absolute values and inequalities we have $-s+a<s_n-s$ which implies $s_n>a$ for $n>N$."
    $endgroup$
    – md2perpe
    yesterday











  • $begingroup$
    That would be just fine!
    $endgroup$
    – José Carlos Santos
    yesterday















2












$begingroup$

I would just add, after “Choose $varepsilon=s-a$”, that there is a $Ninmathbb N$ such that $n>Nimplieslvert s_n-srvert<varepsilon$.






share|cite|improve this answer









$endgroup$












  • $begingroup$
    Something like: "Let $varepsilon=s-a>0$. Then, by definition of limit of sequence, there exists $N in mathbbN$ such that $|s_n-s|<varepsilon=s-a$ whenever $n>N$. By properties of absolute values and inequalities we have $-s+a<s_n-s$ which implies $s_n>a$ for $n>N$."
    $endgroup$
    – md2perpe
    yesterday











  • $begingroup$
    That would be just fine!
    $endgroup$
    – José Carlos Santos
    yesterday













2












2








2





$begingroup$

I would just add, after “Choose $varepsilon=s-a$”, that there is a $Ninmathbb N$ such that $n>Nimplieslvert s_n-srvert<varepsilon$.






share|cite|improve this answer









$endgroup$



I would just add, after “Choose $varepsilon=s-a$”, that there is a $Ninmathbb N$ such that $n>Nimplieslvert s_n-srvert<varepsilon$.







share|cite|improve this answer












share|cite|improve this answer



share|cite|improve this answer










answered yesterday









José Carlos SantosJosé Carlos Santos

166k22132235




166k22132235











  • $begingroup$
    Something like: "Let $varepsilon=s-a>0$. Then, by definition of limit of sequence, there exists $N in mathbbN$ such that $|s_n-s|<varepsilon=s-a$ whenever $n>N$. By properties of absolute values and inequalities we have $-s+a<s_n-s$ which implies $s_n>a$ for $n>N$."
    $endgroup$
    – md2perpe
    yesterday











  • $begingroup$
    That would be just fine!
    $endgroup$
    – José Carlos Santos
    yesterday
















  • $begingroup$
    Something like: "Let $varepsilon=s-a>0$. Then, by definition of limit of sequence, there exists $N in mathbbN$ such that $|s_n-s|<varepsilon=s-a$ whenever $n>N$. By properties of absolute values and inequalities we have $-s+a<s_n-s$ which implies $s_n>a$ for $n>N$."
    $endgroup$
    – md2perpe
    yesterday











  • $begingroup$
    That would be just fine!
    $endgroup$
    – José Carlos Santos
    yesterday















$begingroup$
Something like: "Let $varepsilon=s-a>0$. Then, by definition of limit of sequence, there exists $N in mathbbN$ such that $|s_n-s|<varepsilon=s-a$ whenever $n>N$. By properties of absolute values and inequalities we have $-s+a<s_n-s$ which implies $s_n>a$ for $n>N$."
$endgroup$
– md2perpe
yesterday





$begingroup$
Something like: "Let $varepsilon=s-a>0$. Then, by definition of limit of sequence, there exists $N in mathbbN$ such that $|s_n-s|<varepsilon=s-a$ whenever $n>N$. By properties of absolute values and inequalities we have $-s+a<s_n-s$ which implies $s_n>a$ for $n>N$."
$endgroup$
– md2perpe
yesterday













$begingroup$
That would be just fine!
$endgroup$
– José Carlos Santos
yesterday




$begingroup$
That would be just fine!
$endgroup$
– José Carlos Santos
yesterday

















draft saved

draft discarded
















































Thanks for contributing an answer to Mathematics Stack Exchange!


  • Please be sure to answer the question. Provide details and share your research!

But avoid


  • Asking for help, clarification, or responding to other answers.

  • Making statements based on opinion; back them up with references or personal experience.

Use MathJax to format equations. MathJax reference.


To learn more, see our tips on writing great answers.




draft saved


draft discarded














StackExchange.ready(
function ()
StackExchange.openid.initPostLogin('.new-post-login', 'https%3a%2f%2fmath.stackexchange.com%2fquestions%2f3140507%2fif-a-sequence-s-n-of-real-numbers-converges-to-sa-then-s-na%23new-answer', 'question_page');

);

Post as a guest















Required, but never shown





















































Required, but never shown














Required, but never shown












Required, but never shown







Required, but never shown

































Required, but never shown














Required, but never shown












Required, but never shown







Required, but never shown







Popular posts from this blog

Lowndes Grove History Architecture References Navigation menu32°48′6″N 79°57′58″W / 32.80167°N 79.96611°W / 32.80167; -79.9661132°48′6″N 79°57′58″W / 32.80167°N 79.96611°W / 32.80167; -79.9661178002500"National Register Information System"Historic houses of South Carolina"Lowndes Grove""+32° 48' 6.00", −79° 57' 58.00""Lowndes Grove, Charleston County (260 St. Margaret St., Charleston)""Lowndes Grove"The Charleston ExpositionIt Happened in South Carolina"Lowndes Grove (House), Saint Margaret Street & Sixth Avenue, Charleston, Charleston County, SC(Photographs)"Plantations of the Carolina Low Countrye

random experiment with two different functions on unit interval Announcing the arrival of Valued Associate #679: Cesar Manara Planned maintenance scheduled April 23, 2019 at 00:00UTC (8:00pm US/Eastern)Random variable and probability space notionsRandom Walk with EdgesFinding functions where the increase over a random interval is Poisson distributedNumber of days until dayCan an observed event in fact be of zero probability?Unit random processmodels of coins and uniform distributionHow to get the number of successes given $n$ trials , probability $P$ and a random variable $X$Absorbing Markov chain in a computer. Is “almost every” turned into always convergence in computer executions?Stopped random walk is not uniformly integrable

How should I support this large drywall patch? Planned maintenance scheduled April 23, 2019 at 00:00UTC (8:00pm US/Eastern) Announcing the arrival of Valued Associate #679: Cesar Manara Unicorn Meta Zoo #1: Why another podcast?How do I cover large gaps in drywall?How do I keep drywall around a patch from crumbling?Can I glue a second layer of drywall?How to patch long strip on drywall?Large drywall patch: how to avoid bulging seams?Drywall Mesh Patch vs. Bulge? To remove or not to remove?How to fix this drywall job?Prep drywall before backsplashWhat's the best way to fix this horrible drywall patch job?Drywall patching using 3M Patch Plus Primer